When a patient failed to respond to prescribed medication, the doctor hypothesized that the dosage was insufficient. ...

on November 13, 2019

Help

Could you explain why B and not D?

Replies
Create a free account to read and take part in forum discussions.

Already have an account? log in

on November 13, 2019

I mean why D and not B?

Ben on November 13, 2019

Hi Vijetakanabar90, thanks for the question!

This is an unusual question stem and a rather difficult set of ideas to follow. But let's break it down into simple terms:

The doctor's second recommendation: the doctor advised the patient to resume initial dosage and stop drinking the herbal beverage

The application and result of this: the medication still did not work

Initial hypothesis by the doctor: insufficient dosage

This confirms answer choice D because it shows that the beverage was not the only reason that the initial dosage did not work. We know this because when the doctor tested the initial dosage in the absence of the herbal medication, it still did not work.

Answer choice B is unrelated to the result of the doctor's second recommendation. It states that it is "less plausible that the beverage actually contributed to the ineffectiveness of the prescribed medication". In both cases, the medication didn't work so far, therefore we can't rule out that or make less plausible that the beverage had some impact on this. Only after the final recommendation of doubling the dose without the herbal beverage can we say that the beverage did, in fact, have an impact. To prove that the herbal beverage had no impact would mean that the dose was continually increased and there was still no response.

Hope this helps!

on November 13, 2019

Thank you!